LSAT and Law School Admissions Forum

Get expert LSAT preparation and law school admissions advice from PowerScore Test Preparation.

 Administrator
PowerScore Staff
  • PowerScore Staff
  • Posts: 8937
  • Joined: Feb 02, 2011
|
#73949
Complete Question Explanation

StrengthenX—CE. The correct answer choice is (E)

The conclusion of the argument is based on the causal assumption that
electronic devices cause a disturbance in low-power circuitry, creating an
obvious danger:

..... ED = electronic devices
..... I = interference with low-power circuitry

..... ..... C ..... E
..... ..... ED :arrow: I

The question stem is a StrengthenX (remember, Least works like Except
in question stems) and thus the four incorrect answers will each strengthen
the argument. As you attack the answer choices, look for the five causal
strengthening answer types discussed earlier.

Answer choice (A): This answer choice strengthens the argument by
showing that when the cause is absent, the effect does not occur (Type C).
Once the laptop was turned off, the cause disappeared, and according to
the author’s beliefs, the effect should then disappear as well.

Answer choice (B): This answer strengthens the argument by showing that
the data used to make the conclusion are accurate (Type E). By stating that
all electronic devices emit radiation, the answer choice closes a hole in the
argument.

Answer choice (C): This answer choice strengthens the argument by
showing that when the cause is absent, the effect does not occur (Type C).

Answer choice (D): This answer strengthens the argument by showing that
the data used to make the conclusion are accurate (Type E). By showing
that radiation can travel far enough to reach the cockpit, the cause is
confirmed as possible.

Answer choice (E): This is the correct answer. The fact that the circuitry
and electronic devices became popular at the same time does not offer any
supporting evidence to the contention that the electronic devices cause the
interference with the low power circuitry. This answer has no effect on the
argument and is therefore correct.

Get the most out of your LSAT Prep Plus subscription.

Analyze and track your performance with our Testing and Analytics Package.